nombre harmonique asymptotique

Bonjour,
Auriez-vous une référence pour, ou une preuve rapide de, l'équivalence en $n\to\infty$ $$\log\dfrac{n^a}{\sum_{k=1}^n k^a} \sim -a \log n$$ lorsque $a>0$ ?

Réponses

  • Ça m'a l'air faux :
    $\displaystyle \sum_{k=1}^n k^a = n^{a+1} \dfrac{1}{n} \sum_{k=1}^n \left(\frac{k}{n}\right)^a$. Or, $\displaystyle \dfrac{1}{n} \sum_{k=1}^n \left(\frac{k}{n}\right)^a$ tend vers $\displaystyle \int_0^1 x^a dx = \dfrac{1}{a+1}$ (somme de Riemann).
    Donc $\displaystyle \sum_{k=1}^n k^a \sim \dfrac{n^{a+1}}{a+1}$, puis $\displaystyle \dfrac{n^a}{\sum_{k=1}^n k^a} \sim \dfrac{a+1}{n}$.
    En passant au $\ln$, on trouve que $ \displaystyle \ln\left[ \dfrac{n^a}{\sum_{k=1}^n k^a}\right] \sim -\ln(n)$.
  • $$
    \frac{1}{n}\sum_{k=1}^n \left(\frac{k}{n}\right)^a
    $$
    est une somme de Riemann.

    Edit : grillé par Guego.
  • ...et avec la formule d'Euler-MacLaurin, par exemple, on arrive assez vite à
    $$\log \left ( \frac{n^a}{\sum_{k=1}^n k^a} \right) = - \log n + \log(a+1) - \frac{a+1}{2n} + O_a \left( \frac{1}{n^2} \right).$$
  • Merci à tous. J'essaye de finir mon dernier article puis je vais prendre ma retraite en maths.
Connectez-vous ou Inscrivez-vous pour répondre.